the value of an investment decreased by 10%. The original amount of the investment was $75 what is the current value of the investment?​

Answers

Answer 1

Answer:

67.5

Step-by-step explanation:


Related Questions

To practice for a competition, Alexia swam 0.54 kilometers in the pool each day for 2 weeks. How many meters did Alexia swim in those 2 weeks?

Answers

Answer:Alexia swam  7,560meters in 2 weeks

Step-by-step explanation:

Step 1

7 days = 1 week

therefore 2 weeks = 7 x 2 = 14 days

Step 2

Each day  Alexia swam =  0.54 kilometres

So in 14 days( 2 weeks) She swam = 14 x 0.54 km= 7.56km

Answer is needed in meters

so 1000 m = 1km

Therefore 7.56 km = 7.56km x 1000=7,560meters

Which of the following equations is not a proportion?

A. 2/3 = 142/213
B. 22/33 = 4/6
C. 14/7 = 7/3.5
D. 1.5/3 = 3/9

Answers

Answer:

both of the people above me are wrong, its

22/33=4/6

Step-by-step explanation:

(22/33)=(4/6) is the following equations are not a proportion. Option B is correct.

What is the difference between a ratio and a proportion?

A ratio is an ordered pair of integers a and b expressed as a/b, with b never equaling 0. A percentage is a mathematical expression in which two ratios are specified to be equal.

The two numbers in ratio in numerator and denominator is divided by the same number and obtained as the whole number as;

[tex]\rm \frac{a}{b} = \frac{2 \times a}{2 \times b}[/tex]

(22/33)=(4/6) is the following equations are not a proportion.

Hence, option B is correct.

To learn more about the ratio, refer to the link;

https://brainly.com/question/14335762

#SPJ2

so my kid dosent know this 468 divided by 32

Answers

Answer:

14.625

Step-by-step explanation:

Answer: 14.625

Step-by-step explanation: can i have brainliest?

PLEASE HELP!!!
what is the average rate of change of the equation: y=4x^2-10x+2, between f(3) and f(5)? ​

Answers

Answer:

Average rate of change=22

Step-by-step explanation:

We need to find the average rate of change of the equation: [tex]y=4x^2-10x+2[/tex]between f(3) and f(5).

The formula used to find average rate of change is: [tex]Average \ rate \ of \ change=\frac{f(b)-f(a)}{b-a}[/tex]

In the given question we have a=3 and b =5

Finding f(a) i.e f(3)

[tex]f(3)=4(3)^2-10(3)+2\\f(3)=4(9)-30+2\\f(3)=36-30+2\\f(3)=8[/tex]

Finding f(b0 i.e f(5)

[tex]f(5)=4(5)^2-10(5)+2\\f(5)=4(25)-50+2\\f(5)=100-50+2\\f(5)=52[/tex]

Putting values of f(3)=8 and f(5)=52 to find average rate of change

[tex]Average \ rate \ of \ change=\frac{f(b)-f(a)}{b-a}\\Average \ rate \ of \ change=\frac{52-8}{5-3}\\Average \ rate \ of \ change=\frac{44}{2}\\Average \ rate \ of \ change=22[/tex]

So, Average rate of change=22

Help! Will give brainlest

Answers

Answer:

A. 3, 7, 9, 10, 12

Step-by-step explanation:

All of these numbers as x are less than 14.

Hope it helps!

What is the answer to 8/10 x 5/2

Answers

Answer:

your answer is 2!

Step-by-step explanation:

you have to make it an improper fraction once you do you get 2!

brainliest please>

Answer:

2

Step-by-step explanation:

8/10 = 0.8

5/2 = 2.5

0.8 x 2.5 = 2

QUICK PLEASEEEEEEE!!!!!!!!!!!!?!???!?!?!

what time is it

(plz answr quikly)

Answers

Answer:

it is 10:22am

Step-by-step explanation:

hope it helped brainliest?

A lacrosse team won 80% of their games. If they won 24 games, how many games did they play?

Answers

Answer:

30

Step-by-step explanation:

80% of 30 is 24 (u can use a calculator if you would like to check the answer but it is 30)

Answer: 30 games

Step-by-step explanation:

24 game won 6 game loss and 80%

these 3 questions i need help

Answers

Answer:

1)=-80 | 2)=8,9,10 | 3)=12

Step-by-step explanation:

-80*3/4=-60

8+9+10=27

12*2-4=20

I hope this helps

Please help me i attached a photo

Answers

Answer:

negative twelve

Step-by-step explanation:

the slope is -1, so one must go down as the other goes up. since the x value has increased by 3, the y value must decrease by three

Emma wants to strain a circular deck that has a diameter of 12 m the cost of the wood stain is $3.25 per square meter how much

Answers

Answer:

0.25

Step-by-step explanation:

whats the slope Hurry!

Answers

I think the slope is 2
I think it’s 2 but not sure

Move the digits in 625,134 to create a

new number.

Move the 2 so it is worth 1o as much.

Move the 3 so it is worth 10 times as

much.

Move the 5 so it is worth 50,000.

Move the 4 so its value changes to

4 X 100,000.

Move the 1 and the 6 so that the sum of

their values is 16.

write the new number

Answers

Answer:

a. The new number created is 265,134.

b. The new number created is 625,314.

c. The new number created is 652,314.

d. The new number created is 462,513.

e. The new number created is 253,416.

Step-by-step explanation:

Note: The first question is not correctly and fully stated. It is therefore restated as the questions are answered as follows:

a. Move the 2 so it is worth 10 as much.

From 625,134, the 2 implies 20,000.

If we move the 2 so it is worth 10 as much, it implies that 20,000 is multiplied by 10 and the answer is as follows:

20,000 * 10 = 200,000

The answer implies that 2 becomes the first number and the new number created from 625,134 is as follows:

The new number created is 265,134.

b. Move the 3 so it is worth 10 times as much.

From 625,134, the 3 implies 30.

If we move the 3 so it is worth 10 as much, it implies that 30 is multiplied by 10 and the answer is as follows:

30 * 10 = 300

The answer implies that 3 becomes the fourth number and the new number created from 625,134 is as follows:

The new number created is 625,314.

c. Move the 5 so it is worth 50,000.

This implies that 5 becomes the second number and the new number created from 625,134 is as follows:

The new number created is 652,314.

d. Move the 4 so its value changes to 4 X 100,000

This implies that 4 is now 400,000 and it now becomes the first number. The new number created from 625,134 is as follows:

The new number created is 462,513.

e. Move the 1 and the 6 so that the sum of their values is 16.

This implies the one becoms 10 and the 6 becomes just 6.

As a result, this implies that the 1 now becomes the fifth number and the 6 now become the last number. The new number created from 625,134 is now as follows:

The new number created is 253,416.

what is 3/8 divided by 5/12​

Answers

Answer:

it would be 9/10.

Step-by-step explanation:

-4(-6y – 3) = 12 What is the value of y?

Answers

Answer:

y=0

Step-by-step explanation:

So you would do -4(6y-3) and divide that by -4

then you would do 12 and divide that by -4

or

You could do

-6y-3 which equals -3

then

add three to both sides like this -6y-3+3=-3+3

then simplify it to -6y=0

then divide -6y over -6 and 0 over -6

then it will simplify to 0

YO ANYONE GOT SOME HELLA VIBING SONGS TO JAM OUT TO IM BORREEDD AS HEELLLLLLLLLLLLL

Answers

What type of music do like lol

Cos2A + cos2A cot2A = cot2A​

Answers

cos(2A) + cos(2A) cot(2A) = cot(2A)

cos(2A) + cos(2A) cot(2A) - cot(2A) = 0

cot(2A) sin(2A) + cos(2A) cot(2A) - cot(2A) = 0

cot(2A) (sin(2A) + cos(2A) - 1) = 0

cot(2A) = 0   or   sin(2A) + cos(2A) - 1 = 0

cot(2A) = 0   or   sin(2A) + cos(2A) = 1

cot(2A) = 0   or   sin(2A) cos(π/4) + cos(2A) sin(π/4) = 1/√2

cot(2A) = 0   or   sin(2A + π/4) = 1/√2

[2A = cot⁻¹(0) + ]   or

[2A + π/4 = sin⁻¹(1/√2) + 2   or   2A + π/4 = π - sin⁻¹(1/√2) + 2]

(where n is any integer)

[2A = π/2 + ]   or

[2A + π/4 = π/4 + 2   or   2A + π/4 = 3π/4 + 2]

[A = π/4 + /2]   or

[2A = 2   or   2A = π/2 + 2]

[A = π/4 + /2]   or

[A =   or   A = π/4 + ]

A = (1 + 2n) π/4   or   A =   or   A = π/4 +

A store is having a sale on almonds and jelly beans. For 4 pounds of almonds and 8 pounds of jelly beans, the total cost is $25. For 2 pounds of almonds and 3
pounds of jelly beans, the total cost is $10. Find the cost for each pound of almonds and each pound of jelly beans.

Answers

let x = number of pounds of jelly beans

    y = number of pounds of almonds .

then , 4x + 8y =33

          2x + 3y = 13

multiply the second equation by -2 to get the system :

4x + 8y =33

-4x - 6y = -26

adding the equations ,we get 2y=7. so y=3.5

substitute for y in either of the given equations : 2x + 3 (3.5) =13

+ 10.5=13

     2x =2.5

        x=1.25

answer : A pound of jelly beans costs $1.25 and a pound of almonds costs $3.50.

find the greatest common factor of each set 15,20 ​

Answers

Answer:

5

Step-by-step explanation:

.......................

Please help me
A graph is shown below:

Answers

Answer:

sorry but their is no picture its black...

Answer:

Option D is correct

Step-by-step explanation:

For this question we would need the equation of line since this is a multiple choice question you wouldn't have enough time to find out the equation so we need to be FAST

If we see all the four options at first glance option C or option D seem correct because the line represents a rise in values of x and y respectively, which means x increases with respect to y. So the slope/gradient in other words m should be positive so option A and B are ruled out because the equation of line is written as

[tex]y=mx+b[/tex]

where m is the slope and b is the y-intercept so seeing the graph our equation of line should be

[tex]y=mx-2[/tex]

where b = -2 because value of b corresponds to y-intercept which means the where the graph intersects the y-axis in this case its -2

where m is positive in this case 5 thus

[tex]y=5x-2[/tex]

if we try to rearrange the four choices option C and D give us the above equation and option A and B give us

[tex]y=-5x+2[/tex]

which is wrong !

Now, regarding the inequality sign the graph represents

[tex]y=5x-2[/tex]

and the area which is shaded in blue is the region lower the line which means it should be

[tex]y\leq 5x-2[/tex]

in this case its D if we rearrange it

I have two images so you understand it better visually

As you can see in both forms are shaded region is the same so option D is correct

The Hu family goes out for lunch, and the price of the meal is $44. The sales tax on the meal is 6%, and the family also leaves a 20% tip on the pre-tax amount. What is the total cost of the meal?


The total cost of the meal is $

Answers

Answer:

Step-by-step explanation:

1

find the amount of tax

2

find the amount of the tip

3

find the total cost by adding the cost of the meal, the tax, and the tip

RESULT

56.70

11. A 3000-kg truck moving rightward with a speed of 5 km/hr collides head-on with a 1000-kg car moving leftward with a speed of 10 km/hr. The two vehicles stick together and move with the same velocity after the collision. Determine the post-collision speed of the car and truck.​

Answers

Yo qreo que es un 3000-kg porque eles mal

Determine the missing information in the paragraph proof.

Given: Line PQ is rotated 90° counterclockwise to form line P’Q’. The lines are perpendicular. Line PQ contains the points (a, b) and (c, d). Line P’Q’ contains the points (–b, a) and (–d, c).

Prove: The slopes of perpendicular lines are negative reciprocals.

On a coordinate plane, 2 perpendicular lines are shown. Line P Q has points (a, b) and (c, d). Line P prime Q prime has points (negative b, a) and (negative d, c).

The slopes of lines PQ and P’Q’ can be determined using the formula m = StartFraction v 2 minus v 1 Over x 2 minus x 1 EndFraction

The product of these slopes is ________. This product shows that the slopes are negative reciprocals. It is given that the lines are perpendicular and we have shown that the slopes of the lines are negative reciprocals.

Answers

Answer:

The answer is A.

Step-by-step explanation:

(StartFraction d minus b Over c minus a EndFraction) (StartFraction c minus a Over negative d + b EndFraction) = negative 1

Answer:

A.

Step-by-step explanation:

Edge 2021

What is the best price?
1 bag = $0.89
6 bags = $16.48
48 bags = $44.89
second question: What is the unit price of the best deal? (Round to nearest penny... do not use
symbols)

Answers

Answer:

1 bag

Step-by-step explanation:

because the other ones are more expensive

A train travels at a constant speed for 528 miles. If it takes the train 8 hours to travel the distance, what is
the unit rate at which the train travels?

Answers

Answer:

The train travels at 66 miles per hour.

Step-by-step explanation:

Given that:

Time taken by train = 8 hours

Distance covered by train in 8 hours = 528 miles

Unit rate is defined by the distance travelled by train in one hour.

Therefore, we will divide the total time taken by train to distance covered by train in that time.

Unit rate = [tex]\frac{Distance\ travelled}{Time\ taken}[/tex]

Unit rate = [tex]\frac{528}{8}[/tex]

Unit rate = 66 miles per hour

Hence,

The train travels at 66 miles per hour.

(8+5i) + (10+6i)-(3+6i)

Answers

Answer:

15 + 5i

Step-by-step explanation:

[tex](8+5i) + (10+6i)-(3+6i) \\ \\ = 8 + 5i + 10 + 6i - 3 - 6i \\ \\ =( 8 + 10 - 3 )+ (5i + 6i - 6i )\\ \\ = 15 + 5i[/tex]

In 2001, the world population 6.5 billion and was increasing at a rate of 1.2% each year.
Find the population in 2017. Round to the nearest tenth.

Answers

Answer:

8.0 billion

Step-by-step explanation:

1.2÷100=0.012

0.012×6.5=0.078

2017-2001=16

0.078×16=1.248

6.5+1.248=7.748

nearest tenth 8.0

Need help if correct you get brainliest answer.​

Answers

Answer: the line is intercepting with 4 if that's what you're asking

Step-by-step explanation:

A customer purchases two bags of cherries if each bag costs $7.99 each and He pays with a $20 bill. with no tax, how much change will he receive back?​

Answers

Answer:

12.01

Step-by-step explanation:

20-7.99

Rewrite 2.568 - 1.2 to make the divisor a whole number. Select the answer

choice that shows the correct setup for long division and answer.

O A. 12)2.568 = 0.214

O B. 12) 2.568 = 2.14

O c. 12)

25.68 = 2.41

O D. 12) 25.68 = 2.14

SUBMIT

Answers

Answer:

D. 12) 25.68 = 2.14

Step-by-step explanation:

Given :

Divisor = 1.2

Dividend = 2.568

We can make the divisor a whole number by multiplying the divisor by 10,

i.e. Divisor = 1.2 x 10 = 12

Similarly , dividend = 2.568 x 10 = 25.68

Therefore,

         2.14            

 12 | 25.68

  (-)  24                        

           16

      (-) 12                      

             48

      (-)  48                

             0

Hence the answer is 2.14

Other Questions
Communication that takes place among members of work groups at the same level is known as ________ communication.A) grapevineB) downwardC) upwardD) diagonalE) lateral Please answer I will give brainliest guarantee. what is the Ethical in Media?! explain in your own words After working for 24h Zoe made 234$ after working 40h zoe made 390$ predict how much she will make after 10h of work Which of the following equations would be appropriate to solve:Find three consecutive integers whose sum is 21.A 21a = 3B 3a = 21C)ata+1+a+2 = 21D a +a+a= 21What would the answer choice be? The picture shows to containers filled with a gas, both initially at room temperatureAAAIceHeatWhich statement is correct? (5 points)The gas particles in both containers have the same average kinetic energy because they have equal number of particlesThe gas particles in both containers have the same average kinetic energy because they have the same volumeThe average kinetic energy of the gas particles is greater in container A because it has a lower temperatureThe average kinetic energy of the gas particles is greater in container 8 because its particles move fasterOd Help algebra pls MULTIPLICATION OF POLYNOMIALS What is y-intercept for the line y=8x+21 Which programming language's program structure is similar to its syntax?O A. JavaOB.B. Python. .PHPOD. LISP Niagara Falls is 51 meters high. If you dropped a ball off the ledge, how much time would it take it to hit the water?PLEASE SHOW WORK WILL MARK BRAINLIEST Alfonso runs 10 km at an average speed of x km/h.The next day he runs 12 km at an average speed of (x - 1) km/h.The time taken for the 10km run is 30 minutes less than the time taken for the 12 km run.(a) (i)Write down an equation in x and show that it simplifies to x? 5x 20 = 0. PLEASE HELP URGENT!!!! I need help solving question number 9! How did Greek ideas of democracy shape our ideas of American government today? Blake's pool can hold 2.539 gal of water when full. If it takes 9 hours to fill the empty pool. How much water will be in the pool after 1 hour? The following people have been arrested and charged with a variety of crimes. For each case, decide whether the person should be released and, if so, under what conditions: (1) bail (release after a certain amount of money is paid; set an amount), (2) personal recognizance (release with no money), (3) conditional release (release under certain conditions; set the conditions), or (4) pretrial detention (no release).Name: Marta GarciaAge: 26Charge: Possession of crack cocaineResidence: 619 30th Street; lives alone; no family or references.Employment: UnemployedEducation: 11th gradeCriminal record: As a juvenile, five arrests, mostly misdemeanors. As an adult, two arrests for petty larceny and a conviction for possession of dangerous drugs. Probation was successfully completed.Comment: Arrested while leaving a train station with a large quantity of crack cocaine. Urine test indicates use of narcotics. can u guys pretty please help 6th grade math please help me in all 10 questions. What factors contributed to the military coup in Ghana? PLEASE HELP ME!!!!Who is the target audience for an open letter? A.a small business owner B.a public group a member of Congress C.a private investigator